LSAT and Law School Admissions Forum

Get expert LSAT preparation and law school admissions advice from PowerScore Test Preparation.

User avatar
 Dave Killoran
PowerScore Staff
  • PowerScore Staff
  • Posts: 5853
  • Joined: Mar 25, 2011
|
#88064
Complete Question Explanation
(The complete setup for this game can be found here: lsat/viewtopic.php?f=165&t=7597)

The correct answer choice is (E).

Given that from the first rule L cannot send the first message, and from the second rule the first and last messages must be from the same person, we can infer that L can never send the last message. If there are six messages, the sixth message must be the last one, and so we can infer that L can never send the sixth message. Accordingly, answer choice (E) is correct.

Answer choice (D) is incorrect because the fifth message could be from L, as long as there is a sixth message as well. Template 3A shows that it is possible for L to send the fifth message.

The answers also reveal that there must be at least four messages. The phrase “if there is a fifth/sixth one” appears only after answer choices (D) and (E). Answer choice (C), which specifies a fourth message, does not contain this phrase, which is an indication from the test makers that any solution to this game must contain at least four messages. Although we already identified this fact in our setup, if you had not identified this piece of information, this question could help you make the inference.

Get the most out of your LSAT Prep Plus subscription.

Analyze and track your performance with our Testing and Analytics Package.